Module 2: Ch 16-19, 21

¡Supera tus tareas y exámenes ahora con Quizwiz!

2. The nurse is performing a blood glucose test every 4 hours on an infant born to a diabetic mother. This is to assess the infant's risk of hypoglycemia. The nurse becomes concerned if the infant's blood glucose concentration falls below ______ mg/dl.

ANS: 40 If the newborn has a blood glucose level below 40 mg/dl intervention such as breastfeeding or bottle-feeding should be instituted. If levels remain low after this intervention an intravenous with dextrose may be warranted.

1. The labor and delivery nurse is using a well-known method to quantify the intensity of labor contractions with internal monitoring. This method is known as MVUs, or ______________.

ANS: Montevideo units The baseline intrauterine pressure for each contraction within a 10-minute period is subtracted from the peak pressure. The resulting pressures (peak subtract baseline) are added together to calculate the Montevideo units.

1. Inquiring about past pregnancies is an important part of the nursing assessment. Women who have had a previous cesarean birth may request a trial of labor and a ______ delivery.

ANS: VBAC Although vaginal birth after cesarean is less common, it may be chosen for a variety of reasons. The nurse should be aware of the need for increased support of the woman in labor, and for complications that may occur.

1. Today the pregnant woman has a number of non-pharmacologic measures available to her to assist with pain relief in labor. One such measure involves the application of heat, cold, or pressure to specific areas of the body and is known as _________.

ANS: acupressure The effectiveness of acupressure has been attributed to the gate-control theory of pain and an increase in endorphin levels. Pressure is applied to acupuncture points called tsubos. These points have an increased density of neuroreceptors and increased electrical conductivity. Pressure is initially applied with contractions and then continually as labor progresses.

2. The nurse is caring for a woman in labor at 39 weeks and 5 days of gestation. Her labor progress has slowed due to poor contractions. After discussion with the provider, a decision has been made that she is a good candidate for ___________ of labor.

ANS: augmentation Augmentation of labor with oxytocin is considered when labor has begun spontaneously but progress has slowed or stopped. The rate of oxytocin may be lower than that of an induction.

3. A ________ succedaneum may appear over the vertex of the newborn's head as a result of pressure against the mother's cervix while in utero.

ANS: caput This pressure causes localized edema and appears as an edematous area on the infant's head. The edema may cross suture lines, is soft to the touch, and varies in size. It usually resolves quickly and disappears entirely within the first few days after birth. Caput may also occur as the result of an operative delivery when a vacuum extractor is used during a vaginal birth.

2. Many methods of nonpharmacologic pain control become less effective after prolonged use. This is a process called _________.

ANS: habituation Offering a variety of techniques counters the effects of habituation. The nurse who understands how to apply a number of methods of pain relief can select those that will be most helpful to the individual woman in labor.

1. A popular preinduction cervical ripening agent that is Food and Drug Administration (FDA) approved for the treatment of peptic ulcers is __________.

ANS: misoprostol Cytotec This synthetic prostaglandin tablet is used primarily for the prevention of peptic ulcers. Because of its low cost, stability, and ease of use, many facilities use this medication for cervical ripening and the induction of labor. The manufacturer does not intend to seek FDA approval for other indications; however, ACOG supports its use for these purposes.

1. The shivering mechanism of heat production is rarely functioning in the newborn. Nonshivering _____________ is accomplished primarily by metabolism of brown fat, which is unique to the newborn, and by increased metabolic activity in the brain, heart, and liver.

ANS: thermogenesis Brown fat is located in superficial deposits in the interscapular region and axillae, as well as in deep deposits at the thoracic inlet, along the vertebral column and around the kidneys. Brown fat has a richer vascular and nerve supply than ordinary fat. Heat produced by intense lipid metabolic activity in brown fat can warm the newborn by increasing heat production by as much as 100%.

32. What is the quickest and most common method to obtain neonatal blood for glucose screening 1 hour after birth? a. Puncture the lateral pad of the heel. b. Obtain a sample from the umbilical cord. c. Puncture a fingertip. d. Obtain a laboratory chemical determination.

ANS: A Feedback A A drop of blood obtained by heel stick is the quickest method of glucose screening. The calcaneus bone should be avoided as osteomyelitis may result from injury to the foot. B Most umbilical cords are clamped in the delivery room and are not available for routine testing. C A neonate's fingertips are too fragile to use for this purpose. D Laboratory chemical determination is the most accurate but the lengthiest method.

11. Before the physician performs an external version, the nurse should expect an order for a a. Tocolytic drug b. Contraction stress test (CST) c. Local anesthetic d. Foley catheter

ANS: A Feedback A A tocolytic drug will relax the uterus before and during version, making manipulation easier. B CST is used to determine the fetal response to stress. C A local anesthetic is not used with external version. D The bladder should be emptied, but catheterization is not necessary.

20. A woman at 40 weeks of gestation should be instructed to go to a hospital or birth center for evaluation when she experiences a. A trickle of fluid from the vagina b. Thick pink or dark red vaginal mucus c. Irregular contractions for 1 hour d. Fetal movement

ANS: A Feedback A A trickle of fluid from the vagina may indicate rupture of the membranes requiring evaluation for infection or cord compression. B Bloody show may occur before the onset of true labor. It does not require professional assessment unless the bleeding is pronounced. C This is a sign of false labor and does not require further assessment. D The lack of fetal movement needs further assessment.

33. A new mother states that her infant must be cold because the baby's hands and feet are blue. The nurse explains that this is a common and temporary condition called a. Acrocyanosis b. Erythema neonatorum c. Harlequin color d. Vernix caseosa

ANS: A Feedback A Acrocyanosis, or the appearance of slightly cyanotic hands and feet, is caused by vasomotor instability, capillary stasis, and a high hemoglobin level. Acrocyanosis is normal and appears intermittently over the first 7 to 10 days. B Erythema toxicum (also called erythema neonatorum) is a transient newborn rash that resembles flea bites. C The harlequin sign is a benign, transient color change in newborns. Half of the body is pale, and the other half is ruddy or bluish red with a line of demarcation. D Vernix caseosa is a cheeselike, whitish substance that serves as a protective covering.

8. The nurse providing newborn stabilization must be aware that the primary side effect of maternal narcotic analgesia in the newborn is a. Respiratory depression b. Bradycardia c. Acrocyanosis d. Tachypnea

ANS: A Feedback A An infant delivered within 1 to 4 hours of maternal analgesic administration is at risk for respiratory depression from the sedative effects of the narcotic. B Bradycardia is not the anticipated side effect of maternal analgesics. C Acrocyanosis is an expected finding in a newborn and is not related to maternal analgesics. D The infant who is having a side effect to maternal analgesics normally would have a decrease in respirations, not an increase.

9. The nurse should alert the physician when a. The infant is dusky and turns cyanotic when crying. b. Acrocyanosis is present at age 1 hour. c. The infant's blood glucose is 45 mg/dL. d. The infant goes into a deep sleep at age 1 hour.

ANS: A Feedback A An infant who is dusky and becomes cyanotic when crying is showing poor adaptation to extrauterine life. B Acrocyanosis is an expected finding during the early neonatal life. C This is within normal range for a newborn. D Infants enter the period of deep sleep when they are about 1 hour old.

4. Excessive anxiety in labor heightens the woman's sensitivity to pain by increasing a. Muscle tension b. Blood flow to the uterus c. The pain threshold d. Rest time between contractions

ANS: A Feedback A Anxiety and fear increase muscle tension, diverting oxygenated blood to the woman's brain and skeletal muscles. Prolonged tension results in general fatigue, increased pain perception, and reduced ability to use coping skills. B Anxiety can decrease blood flow to the uterus. C Anxiety will decrease the pain threshold. D Anxiety will decrease the amount of rest the mother gets between contractions.

12. What occurrence is associated with cervical dilation and effacement? a. Bloody show b. False labor c. Lightening d. Bladder distention

ANS: A Feedback A As the cervix begins to soften, dilate, and efface, expulsion of the mucous plug that sealed the cervix during pregnancy occurs. This causes rupture of small cervical capillaries. B Cervical dilation and effacement do not occur with false labor. C Lightening is the descent of the fetus toward the pelvic inlet before labor. D Bladder distention occurs when the bladder is not empted frequently. It may slow down the decent of the fetus during labor.

4. Nurses can prevent evaporative heat loss in the newborn by a. Drying the baby after birth and wrapping the baby in a dry blanket b. Keeping the baby out of drafts and away from air conditioners c. Placing the baby away from the outside wall and the windows d. Warming the stethoscope and nurse's hands before touching the baby

ANS: A Feedback A Because the infant is a wet with amniotic fluid and blood, heat loss by evaporation occurs quickly. B Heat loss by convection occurs when drafts come from open doors and air currents created by people moving around. C If the heat loss is caused by placing the baby near cold surfaces or equipment, it is termed a radiation heat loss. D Conduction heat loss occurs when the baby comes in contact with cold objects or surfaces.

24. What is the first type of breathing technique used in labor? a. Slow-paced b. Modified-paced c. Patterned-paced d. Pant-blow

ANS: A Feedback A Breathing for the first stage of labor consists of a cleansing breath and various breathing techniques known as paced breathing. The first type used in labor is the slow-paced. B Modified-paced breathing is used when the slow-paced breathing is no longer effective. C Patterned-paced breathing is used later in the labor and has the woman focusing on a pattern of breathing. D Pant-blow breathing can be used to prevent pushing before the cervix is completely dilated.

5. Which patient status is an acceptable indication for serial oxytocin induction of labor? a. Past 42 weeks' gestation b. Multiple fetuses c. Polyhydramnios d. History of long labors

ANS: A Feedback A Continuing a pregnancy past the normal gestational period is likely to be detrimental to fetal health. B Multiple fetuses overdistend the uterus, making induction of labor high risk. C Polyhydramnios overdistends the uterus, making induction of labor high risk. D History of rapid labors is a reason for induction of labor because of the possibility that the baby would otherwise be born in uncontrolled circumstances.

22. An important part of fetal surveillance is assessment and documentation of the fetal heart rate during the first stage of labor. In the low-risk patient assessments for variability and periodic changes if using the fetal monitor should be done a. Every 15-30 minutes b. Every 5-15 minutes c. Every 30-60 minutes d. Only before and after ambulation

ANS: A Feedback A During the active first stage of labor, FHR should be assessed every 15-30 minutes just after a contraction. B During the second stage of labor the FHR should be assessed every 5-15 minutes. C This is not an adequate assessment during any stage of labor. D The FHR should also be evaluated both before and during ambulation.

3. Which mechanism of labor occurs when the largest diameter of the fetal presenting part passes the pelvic inlet? a. Engagement b. Extension c. Internal rotation d. External rotation

ANS: A Feedback A Engagement occurs when the presenting part fully enters the pelvic inlet. B Extension occurs when the fetal head meets resistance from the tissues of the pelvic floor and the fetal neck stops under the symphysis. This causes the fetal head to extend. C Internal rotation occurs when the fetus enters the pelvic inlet. The rotation allows the longest fetal head diameter to conform to the longest diameter of the maternal pelvis. D External rotation occurs after the birth of the head. The head then turns to the side so the shoulders can internally rotate and are positioned with their transverse diameter in the anteroposterior diameter of the pelvic outlet.

21. While assisting with a vacuum extraction birth, what should the nurse immediately report to the physician? a. Persistent fetal bradycardia below 100 bpm b. Maternal pulse rate of 100 bpm c. Maternal blood pressure of 120/70 mm Hg d. Decrease in intensity of uterine contractions

ANS: A Feedback A Fetal bradycardia may indicate fetal distress and may require immediate intervention. B Maternal pulse rate may increase due to the pushing process. C This blood pressure is within expected norms for this stage of labor. D The birth is imminent at this point.

17. Immediately after the forceps-assisted birth of an infant, the nurse should a. Assess the infant for signs of trauma. b. Give the infant prophylactic antibiotics. c. Apply a cold pack to the infant's scalp. d. Measure the circumference of the infant's head.

ANS: A Feedback A Forceps delivery can result in local irritation, bruising, or lacerations of the fetal scalp. B Prophylactic antibiotics are not necessary with a forceps delivery. C This would put the infant at risk for cold stress and would be contraindicated. D Measuring the circumference of the head is part of the initial nursing assessment.

19. What is an essential part of nursing care for the laboring woman? a. Helping the woman manage the pain. b. Eliminating the pain associated with labor. c. Sharing personal experiences regarding labor and delivery to decrease her anxiety. d. Feeling comfortable with the predictable nature of intrapartal care.

ANS: A Feedback A Helping a woman manage the pain is an essential part of nursing care, because pain is an expected part of normal labor and cannot be fully relieved. B Labor pain cannot be fully relieved. C Decreasing anxiety is important, but managing pain is a top priority. D The labor nurse should always be assessing for unpredictable occurrences.

16. Leopold's maneuvers are used by practitioners to determine a. The best location to assess the fetal heart rate (FHR) b. Cervical dilation and effacement c. Whether the fetus is in the posterior position d. The status of the membranes

ANS: A Feedback A Leopold's maneuvers are often performed before assessing the FHR. These maneuvers help identify the best location to obtain the FHR. B Dilation and effacement are best determined by vaginal examination. C Assessment of fetal position is more accurate with vaginal examination. D A Nitrazine or ferning test can be performed to determine the status of the fetal membranes.

34. The nurse notes that a woman who has given birth 1 hour ago is touching her infant with the fingertips and talking to him softly in high-pitched tones. On the basis of this observation, the nurse should a. Document this evidence of normal early maternal-infant attachment behavior. b. Observe for other signs that the mother may not be accepting of the infant. c. Request a social service consult for psychosocial support. d. Determine whether the mother is too fatigued to interact normally with her infant.

ANS: A Feedback A Normal early maternal-infant behaviors are tentative and include fingertip touch, eye contact, and using a high-pitched voice when talking to the infant. B These are signs of normal attachment behavior; no other assessment is necessary at this point. C There is no indication at this point that social service consult is necessary. The signs are of normal attachment behavior. D The mother may be fatigued but is interacting with the infant in an expected manner.

6. Perinatal nurses are legally responsible for a. Correctly interpreting FHR patterns, initiating appropriate nursing interventions, and documenting the outcomes b. Greeting the patient on arrival, assessing her, and starting an IV line c. Applying the external fetal monitor and notifying the care provider d. Making sure the woman is comfortable

ANS: A Feedback A Nurses who care for women during childbirth are legally responsible for correctly interpreting FHR patterns, initiating appropriate nursing interventions based on those patterns, and documenting the outcomes of those interventions. B These activities should be performed when any patient arrives to the maternity unit. The nurse is not the only one legally responsible for performing these functions. C This is a nursing function that is part of the standard of care for all obstetrical patients. This falls within the RN scope of practice. D Everyone caring for the pregnant woman should ensure that both she and her support partner are comfortable.

28. Which nursing action is designed to avoid unnecessary heat loss in the newborn? a. Place a blanket over the scale before weighing the infant. b. Maintain room temperature at 70° F. c. Undress the infant completely for assessments so they can be finished quickly. d. Take the rectal temperature every hour to detect early changes.

ANS: A Feedback A Padding the scale prevents heat loss from the infant to a cold surface by conduction. B Room temperature should be appropriate to prevent heat loss from convection. Also, if the room is warm enough, radiation will assist in maintaining body heat. C Undressing the infant completely will expose the child to cooler room temperatures and cause a drop in body temperature due to convection. D Hourly assessments are not necessary for a normal newborn with a stable temperature.

7. In fetal circulation, the pressure is greatest in the a. Right atrium b. Left atrium c. Hepatic system d. Pulmonary veins

ANS: A Feedback A Pressure in fetal circulation is greatest in the right atrium, which allows a right-to-left shunting that aids in bypassing the lungs during intrauterine life. B The pressure increases in the left atrium after birth and will close the foramen ovale. C The liver does not filter the blood during fetal life until the end. It is functioning by birth. D Blood bypasses the pulmonary vein during fetal life.

18. Which statement correctly describes the nurse's responsibility related to electronic monitoring? a. Teach the woman and her support person about the monitoring equipment and discuss any questions they have. b. Report abnormal findings to the physician before initiating corrective actions. c. Inform the support person that the nurse will be responsible for all comfort measures when the electronic equipment is in place. d. Document the frequency, duration, and intensity of contractions measured by the external device.

ANS: A Feedback A Teaching is an essential part of the nurse's role. B Corrective actions should be initiated first in order to correct abnormal findings as quickly as possible. C The support person should still be encouraged to assist with the comfort measures. D Electronic monitoring will record the contractions and FHR response.

22. Which newborn reflex is elicited by stroking the lateral sole of the infant's foot from the heel to the ball of the foot? a. Babinski b. Tonic neck c. Stepping d. Plantar grasp

ANS: A Feedback A The Babinski reflex causes the toes to flare outward and the big toe to dorsiflex. B The tonic neck reflex (also called the fencing reflex) refers to the posture assumed by newborns when in a supine position. C The stepping reflex occurs when infants are held upright with their heel touching a solid surface and the infant appears to be walking. D Plantar grasp reflex is similar to the palmar grasp reflex: when the area below the toes are touched, the infant's toes curl over the nurse's finger.

24. The nurse auscultates the fetal heart rate (FHR) and determines a rate of 152. Which nursing intervention is appropriate? a. Inform the mother that the rate is normal. b. Reassess the FHR in 5 minutes because the rate is too high. c. Report the FHR to the physician or nurse-midwife immediately. d. Tell the mother that she is going to have a boy because the heart rate is fast.

ANS: A Feedback A The FHR is within the normal range, so no other action is indicated at this time. B The FHR is within the expected range; reassessment should occur, but not in 5 minutes. C The FHR is within the expected range; no further action is necessary at this point. D The sex of the baby cannot be determined by the FHR.

13. The fetal heart rate baseline increases 15 beats per minute after vibroacoustic stimulation. The best interpretation of this is that the fetus is showing a. A reassuring response b. Progressive acidosis c. Parasympathetic stimulation d. A worsening hypoxia

ANS: A Feedback A The fetus with adequate reserve for the stress of labor will usually respond to vibroacoustic stimulation with a temporary increase in the fetal heart rate (FHR) over baseline of 15 bpm for 15 seconds or more. B An increase in the FHR after stimulation is reassuring. C An increase in the FHR after stimulation is a reassuring pattern and does not indicate problems with the parasympathetic nervous system. D An increase in the FHR with stimulation does not indicate hypoxia.

36. To adequately care for a laboring woman, the nurse should know that the _____ stage of labor varies the most in length. a. First b. Second c. Third d. Fourth

ANS: A Feedback A The first stage of labor is considered to last from the onset of regular uterine contractions to full dilation of the cervix. The first stage is much longer than the second and third stages combined. In a first pregnancy, the first stage of labor can take up to 20 hours. B The second stage of labor lasts from the time the cervix is fully dilated to the birth of the fetus. The average length is 20 minutes for a multiparous woman and 50 minutes for a nulliparous woman. C The third stage of labor lasts from the birth of the fetus until the placenta is delivered. This stage may be as short as 3 minutes or as long as 1 hour. D The fourth stage of labor, recovery, lasts about 2 hours after delivery of the placenta.

7. The best time to teach nonpharmacologic pain control methods to an unprepared laboring woman is during which phase? a. Latent phase b. Active phase c. Transition phase d. Second stage

ANS: A Feedback A The latent phase of labor is the best time for intrapartum teaching, because the woman is usually anxious enough to be attentive, yet comfortable enough to understand the teaching. B During the active phase, the woman is focused internally and unable to concentrate on teaching. C During transition, the woman is focused on keeping control; she is unable to focus on anyone else or learn at this time. D During the second stage, the woman is focused on pushing. She normally handles the pain better at this point because she is active in doing something to hasten the delivery.

10. In which situation is a baseline fetal heart rate of 160 to 170 beats per minute be considered a normal finding? a. The fetus is at 28 weeks of gestation. b. The mother has been given an epidural block. c. The mother has a history of fast labors. d. The mother has mild preeclampsia but is not in labor.

ANS: A Feedback A The normal preterm fetus may have a baseline rate slightly higher than the term fetus because of an immature parasympathetic nervous system that does not yet exert a slowing effect on the fetal heart rate (FHR). B Any change in the FHR with an epidural is not considered an expected outcome. C Fast labors should not alter the FHR normally. D Preeclampsia should not cause a normal elevation of the FHR.

19. The most important nursing intervention for the patient who has received an epidural narcotic is a. Monitoring respiratory rate hourly b. Administering analgesics as needed c. Monitoring blood pressure every 4 hours d. Assessing the level of anesthesia

ANS: A Feedback A The possibility of respiratory depression exists for up to 24 hours after administration of an epidural narcotic. B Epidural narcotic should be enough pain relief that further medication is not necessary. Administering any other narcotic may cause an overdose. C The patient's blood pressure needs to be monitored. However, that is not the major concern with this medication. D The epidural narcotic should provide pain relief, but not anesthesia.

22. A newborn infant weighing 8 lb (3632 g) needs naloxone (Narcan). This infant should receive approximately _____ mg. a. 0.36 b. 3.6 c. 0.03 d. 0.3

ANS: A Feedback A The recommended neonatal dose of naloxone is 0.1 mg/kg. B This dose is too large for this infant. C This dose is too small and would not be effective. D This dose is too small.

21. To provide optimal care to the intrapartum woman, the nurse understands that the least favorable maternal position for labor is a. Supine b. Sitting c. Lying on the side d. Standing

ANS: A Feedback A The supine position allows the heavy uterus to compress the inferior vena cava and can reduce placental blood flow, compromising fetal oxygen supply. B The sitting position allows gravity to assist in the descent of the fetus. C The side-lying position is comfortable for most mothers, and it allows for adequate placental blood flow. D Standing and walking may be comfortable for some women. This does not prevent adequate placental blood flow and can be encouraged if there are no other contraindications.

5. The nurse knows that proper placement of the tocotransducer for electronic fetal monitoring is a. Over the uterine fundus b. On the fetal scalp c. Inside uterus d. Over the mother's lower abdomen

ANS: A Feedback A The tocotransducer monitors uterine activity and should be placed over the fundus, where the most intensive uterine contractions occur. B The tocotransducer monitors uterine contractions. C The tocotransducer is for external use. D The most intensive uterine contractions occur at the fundus; this is the best placement area.

13. To be aware of potential risks to the laboring woman, the nurse understands that a breech presentation is associated with a. Umbilical cord compression b. More rapid labor c. A high risk of infection d. Maternal perineal trauma

ANS: A Feedback A The umbilical cord can be compressed between the fetal body and the maternal pelvis when the body has been born but the head remains within the pelvis. B Breech presentation is not associated with a more rapid labor. C There is no higher risk of infection with a breech birth. D There is no higher risk for perineal trauma with a breech birth.

25. A laboring woman is lying in the supine position. The most appropriate nursing action is to a. Ask her to turn to one side. b. Elevate her feet and legs. c. Take her blood pressure. d. Determine if fetal tachycardia is present.

ANS: A Feedback A The woman's supine position may cause the heavy uterus to compress her inferior vena cava, reducing blood return to her heart and reducing placental blood flow. This problem is relieved by having her turn onto her side. B Elevating her legs will not relieve the pressure from the inferior vena cava. C This position may produce hypotension in the woman, but the action should be to prevent this from happening, not to assess for the problem. D If the woman is allowed to stay in the supine position and blood flow to the placental is reduced significantly, fetal tachycardia may occur. The most appropriate nursing action is to prevent this from occurring by turning the woman to her side.

35. A first-time father is changing the diaper of his 1-day-old daughter. He asks the nurse, "What is this black, sticky stuff in her diaper?" The nurse's best response is a. "That's meconium, which is your baby's first stool. It's normal." b. "That's transitional stool." c. "That means your baby is bleeding internally." d. "Oh, don't worry about that. It's okay."

ANS: A Feedback A This is an accurate statement and the most appropriate response. B Transitional stool is greenish brown to yellowish brown and usually appears by the third day after initiation of feeding. C This statement is not accurate. D This statement is not appropriate. It is belittling to the father and does not educate him about the normal stool patterns of his daughter.

2. In order to help patients manage discomfort and pain during labor, nurses should be aware that a. The predominant pain of the first stage of labor is the visceral pain located in the lower portion of the abdomen. b. Somatic pain is the extreme discomfort between contractions. c. The somatic pain of the second stage of labor is more generalized and related to fatigue. d. Pain during the third stage is a somewhat milder version of the second stage.

ANS: A Feedback A This pain comes from cervical changes, distention of the lower uterine segment, and uterine ischemia. B Somatic pain is a faster, sharp pain. Somatic pain is most prominent during late first-stage labor and during second-stage labor as the descending fetus puts direct pressure on maternal tissues. C Second-stage labor pain is intense, sharp, burning, and localized. D Third-stage labor pain is similar to that of the first stage.

38. The cheeselike, whitish substance that fuses with the epidermis and serves as a protective coating is called a. Vernix caseosa b. Surfactant c. Caput succedaneum d. Acrocyanosis

ANS: A Feedback A This protection is needed because the infant's skin is so thin. B Surfactant is a protein that lines the alveoli of the infant's lungs. C Caput succedaneum is the swelling of the tissue over the presenting part of the fetal head. D Acrocyanosis is cyanosis of the hands and feet, resulting in a blue coloring.

2. Which maternal condition is considered a contraindication for the application of internal monitoring devices? a. Unruptured membranes b. Cervix is dilated to 4 cm c. External monitors are currently being used d. Fetus has a known heart defect

ANS: A Feedback A To apply internal monitoring devices, the membranes must be ruptured. B Cervical dilation of 4 cm permits the insertion of fetal scalp electrodes and intrauterine catheter. C The external monitor can be discontinued after the internal ones are applied. D A compromised fetus should be monitored with the most accurate monitoring devices.

20. The nurse caring for the woman in labor should understand that absent or minimal variability is classified as either abnormal or indeterminate. Which condition related to decreased variability is considered benign? a. A periodic fetal sleep state b. Extreme prematurity c. Fetal hypoxemia d. Pre-existing neurologic injury

ANS: A Feedback A When the fetus is temporarily in a sleep state there is minimal variability present. Periodic fetal sleep states usually last no longer than 30 minutes. B A woman who presents in labor with extreme prematurity may display a FHR pattern of minimal or absent variability. C Abnormal variability may also be related to fetal hypoxemia and metabolic acidemia. D Congenital anomalies or pre-existing neurologic injury may also present as absent or minimal variability. Other possible causes might be CNS depressant medications, narcotics or general anesthesia.

1. While developing an intrapartum care plan for the patient in early labor, it is important that the nurse recognize that psychosocial factors may influence a woman's experience of pain. These include (select all that apply) a. Culture b. Anxiety and fear c. Previous experiences with pain d. Intervention of caregivers e. Support systems

ANS: A, B, C, E Feedback Correct Culture: a woman's sociocultural roots influence how she perceives, interprets, and responds to pain during childbirth. Some cultures encourage loud and vigorous expressions of pain, whereas others value self-control. The nurse should avoid praising some behaviors (stoicism) while belittling others (noisy expression). Anxiety and fear: extreme anxiety and fear magnify sensitivity to pain and impair a woman's ability to tolerate it. Anxiety and fear increase muscle tension in the pelvic area, which counters the expulsive forces of uterine contractions and pushing efforts. Previous experiences with pain:fear and withdrawal are a natural response to pain during labor. Learning about these normal sensations ahead of time helps a woman suppress her natural reactions of fear regarding the impending birth. If a woman previously had a long and difficult labor, she is likely to be anxious. She may also have learned ways to cope and may use these skills to adapt to the present labor experience. Support systems: an anxious partner is less able to provide help and support to a woman during labor. A woman's family and friends can be an important source of support if they convey realistic and positive information about labor and delivery. Incorrect Although this may be necessary for the well-being of the woman and her fetus, some interventions add discomfort to the natural pain of labor (i.e., fetal monitor straps).

1. The nurse who elects to practice in the area of obstetrics often hears discussion regarding the "four Ps." These are the four major factors that interact during normal childbirth. What are the "four Ps"? a. Powers b. Passage c. Position d. Passenger e. Psyche

ANS: A, B, D, E Feedback Correct Powers: the two powers of labor are uterine contractions and pushing efforts. During the first stage of labor through full cervical dilation, uterine contractions are the primary force moving the fetus through the maternal pelvis. At some point after full dilation, the woman adds her voluntary pushing efforts to propel the fetus through the pelvis. Passage: the passage for birth of the fetus consists of the maternal pelvis and its soft tissues. The bony pelvis is more important to the successful outcome of labor, because bones and joints do not yield as readily to the forces of labor. Passenger: this is the fetus plus the membranes and placenta. Fetal lie, attitude, presentation, and position are all factors that affect the fetus as passenger. Psyche: the psyche is a crucial part of childbirth. Marked anxiety, fear, or fatigue decreases the woman's ability to cope. Incorrect Position is not one of the "four Ps."

2. Approximately 60% to 80% of women with one low transverse uterine incision from a previous cesarean birth may have a successful vaginal delivery. Recommendations from ACOG related to VBAC risks include (select all that apply) a. Immediate availability of the obstetric provider b. Delivery at a tertiary care center c. Availability of anesthesia personnel d. Personnel who can assist with the cesarean birth e. Use of misoprostol for cervical ripening

ANS: A, C, D Feedback Correct A VBAC delivery should only be attempted with the obstetric provider in house, and anesthesia along with operative personnel readily available to perform a cesarean birth. Incorrect VBAC deliveries may be done in community hospitals if appropriate policies and guidelines for care are in place. Misoprostol administration is contraindicated in a patient with a previous uterine scar.

1. Induction of labor is considered an acceptable obstetric procedure if it is a safe time to deliver the fetus. The charge nurse on the labor and delivery unit is often asked to schedule patients for this procedure and therefore must be cognizant of the specific conditions appropriate for labor induction, including which of the following? Select all that apply. a. Rupture of membranes at or near term b. Convenience of the woman or her physician c. Chorioamnionitis (inflammation of the amniotic sac) d. Postterm pregnancy e. Fetal death

ANS: A, C, D, E Feedback Correct A, C, D, E. These are all acceptable indications for induction. Other conditions include intrauterine growth retardation (IUGR), maternal-fetal blood incompatibility, hypertension, and placental abruption. Incorrect B. Elective inductions for convenience of the woman or her provider are not recommended; however, they have become common. Factors such as rapid labors and living a long distance from a health care facility may be a valid reason in such a circumstance.

27. During the active phase of labor, the FHR of a low-risk patient should be assessed every a. 15 minutes b. 30 minutes c. 45 minutes d. 1 hour

ANS: B Feedback A 15-minute assessments are appropriate for a fetus at high risk. B For the fetus at low risk for complications, guidelines for frequency of assessments are at least every 30 minutes during the active phase of labor. C 45-minute assessments during the active phase of labor is not frequent enough to monitor for complications. D 1-hour assessments during the active phase of labor are not frequent enough to monitor for complications.

10. The nerve block used in labor that provides anesthesia to the lower vagina and perineum is called a(n) a. Epidural b. Pudendal c. Local d. Spinal block

ANS: B Feedback A An epidural provides anesthesia for the uterus, perineum, and legs. B A pudendal block anesthetizes the lower vagina and perineum to provide anesthesia for an episiotomy and use of low forceps if needed. C A local provides anesthesia for the perineum at the site of the episiotomy. D A spinal block provides anesthesia for the uterus, perineum, and down the legs.

7. To adequately teach patients about the process of labor, the nurse knows that which event is the best indicator of true labor? a. Bloody show b. Cervical dilation and effacement c. Fetal descent into the pelvic inlet d. Uterine contractions every 7 minutes

ANS: B Feedback A Bloody show can occur before true labor. B The conclusive distinction between true and false labor is that contractions of true labor cause progressive change in the cervix. C Fetal descent can occur before true labor. D False labor may have contractions that occur this frequently, but it is usually inconsistent.

23. Infants in whom cephalhematomas develop are at increased risk for a. Infection b. Jaundice c. Caput succedaneum d. Erythema toxicum

ANS: B Feedback A Cephalhematomas do not increase the risk for infections. B Cephalhematomas are characterized by bleeding between the bone and its covering, the periosteum. Because of the breakdown of the red blood cells within a hematoma, the infants are at greater risk for jaundice. C Caput is an edematous area on the head from pressure against the cervix. D Erythema toxicum is a benign rash of unknown cause that consists of blotchy red areas.

14. The primary difference between the labor of a nullipara and that of a multipara is the a. Amount of cervical dilation b. Total duration of labor c. Level of pain experienced d. Sequence of labor mechanisms

ANS: B Feedback A Cervical dilation is the same for all labors. B Multiparas usually labor more quickly than nulliparas, making the total duration of their labor shorter. C Level of pain is individual to the woman, not to the number of labors she has experienced. D The sequence of labor mechanisms is the same with all labors.

36. By knowing about variations in infants' blood count, nurses can explain to their patients that a. A somewhat lower than expected red blood cell count could be the result of delay in clamping the umbilical cord. b. The early high white blood cell count (WBC) is normal at birth and should decrease rapidly. c. Platelet counts are higher than in adults for a few months. d. Even a modest vitamin K deficiency means a problem with the blood's ability to clot properly.

ANS: B Feedback A Delayed clamping of the cord results in an increase in hemoglobin and the red blood cell count. B The WBC is high the first day of birth and then declines rapidly. C The platelet count essentially is the same for newborns and adults. D Clotting is sufficient to prevent hemorrhage unless the vitamin K deficiency is significant.

34. The parents of a newborn ask the nurse how much the newborn can see. The parents specifically want to know what type of visual stimuli they should provide for their newborn. The nurse responds to the parents by telling them a. "Infants can see very little until about 3 months of age." b. "Infants can track their parent's eyes and can distinguish patterns; they prefer complex patterns." c. "The infant's eyes must be protected. Infants enjoy looking at brightly colored stripes." d. "It's important to shield the newborn's eyes. Overhead lights help them see better."

ANS: B Feedback A Development of the visual system continues for the first 6 months of life. Visual acuity is difficult to determine, but the clearest visual distance for the newborn appears to be 19 cm. B This is an accurate statement. C Infants prefer to look at complex patterns, regardless of the color. D Infants prefer low illumination and withdraw from bright light.

25. Plantar creases should be evaluated within a few hours of birth because a. The newborn has to be footprinted. b. As the skin dries, the creases will become more prominent. c. Heel sticks may be required. d. Creases will be less prominent after 24 hours.

ANS: B Feedback A Footprinting will not interfere with the creases. B As the infant's skin begins to dry, the creases will appear more prominent, and the infant's gestation could be misinterpreted. C Heel sticks will not interfere with the creases. D The creases will appear more prominent after 24 hours.

30. A 25-year-old primigravida is in the first stage of labor. She and her husband have been holding hands and breathing together through each contraction. Suddenly the woman pushes her husband's hand away and shouts, "Don't touch me!" This behavior is most likely a. Normal and related to hyperventilation b. Common during the transition phase of labor c. A sign that she needs analgesia d. Indicative of abnormal labor

ANS: B Feedback A Hyperventilation will produce signs of respiratory alkalosis. B The transition phase of labor is often associated with an abrupt change in behavior, including increased anxiety and irritability. C If she is in the transitional phase of labor, analgesia may not be appropriate if the birth is near. D This change of behavior is an expected occurrence during the transition phase.

8. Why is continuous electronic fetal monitoring usually used when oxytocin is administered? a. The mother may become hypotensive. b. Uteroplacental exchange may be compromised. c. Maternal fluid volume deficit may occur. d. Fetal chemoreceptors are stimulated.

ANS: B Feedback A Hypotension is not a common side effect of oxytocin. B The uterus may contract more firmly, and the resting tone may be increased with oxytocin use. This response reduces entrance of freshly oxygenated maternal blood into the intervillous spaces, depleting fetal oxygen reserves. C All laboring women are at risk for fluid volume deficit; oxytocin administration does not increase the risk. D Oxytocin affects the uterine muscles.

23. To assist the woman after delivery of the infant, the nurse knows that the blood patch is used after spinal anesthesia to relieve a. Hypotension b. Headache c. Neonatal respiratory depression d. Loss of movement

ANS: B Feedback A Hypotension is prevented by increasing fluid volume before the procedure. B The subarachnoid block may cause a postspinal headache due to loss of cerebrospinal fluid from the puncture in the dura. When blood is injected into the epidural space in the area of the dural puncture, it forms a seal over the hole to stop leaking of cerebrospinal fluid. C Neonatal respiratory depression is not an expected outcome with spinal anesthesia. D Loss of movement is an expected outcome of spinal anesthesia.

2. Which statement is the best rationale for assessing maternal vital signs between contractions? a. During a contraction, assessing fetal heart rates is the priority. b. Maternal circulating blood volume increases temporarily during contractions. c. Maternal blood flow to the heart is reduced during contractions. d. Vital signs taken during contractions are not accurate.

ANS: B Feedback A It is important to monitor fetal response to contractions, but the question is concerned with the maternal vital signs. B During uterine contractions, blood flow to the placenta temporarily stops, causing a relative increase in the mother's blood volume, which in turn temporarily increases blood pressure and slows pulse. C Maternal blood flow is increased during a contraction. D Vital signs are altered by contractions but are considered accurate for that period of time.

25. When instructing the woman in early labor, the nurse teaches her that an important aspect of proper breathing technique is a. Breathing no more than three times the normal rate b. Beginning and ending with a cleansing breath c. Holding the breath no longer than 10 seconds d. Adhering exactly to the techniques as they were taught

ANS: B Feedback A It is important to prevent hyperventilation; however, the cleansing breaths are the most important aspect of the breathing techniques. B The cleansing breath helps the woman clear her mind to focus on relaxing and signals the coach that the contraction is beginning or ending. C The woman should hold her breath for no more than 6 to 8 seconds. D The woman needs to be flexible and change her breathing techniques as needed to keep her comfortable.

7. Which is the most appropriate method of intrapartum fetal monitoring when a woman has a history of hypertension during pregnancy? a. Continuous auscultation with a fetoscope b. Continuous electronic fetal monitoring c. Intermittent assessment with a Doppler transducer d. Intermittent electronic fetal monitoring for 15 minutes each hour

ANS: B Feedback A It is not practical to provide continuous auscultation with a fetoscope. B Maternal hypertension may reduce placental blood flow through vasospasm of the spiral arteries. Reduced placental perfusion is best assessed with continuous electronic fetal monitoring to identify patterns associated with this condition. C This fetus needs continuous monitoring because it is at high risk for complications. D This fetus needs continuous monitoring because it is at high risk for complications.

2. When teaching parents about their newborn's transition to extrauterine life, the nurse explains which organs are nonfunctional during fetal life. They are the a. Kidneys and adrenals b. Lungs and liver c. Eyes and ears d. Gastrointestinal system

ANS: B Feedback A Kidneys and adrenals function during fetal life. The fetus continuously swallows amniotic fluid, which is filtered through the kidneys. B Most of the fetal blood flow bypasses the nonfunctional lungs and liver. C Near term, the eyes are open and the fetus can hear. D The gastrointestinal system functions during fetal life.

7. An indication for an episiotomy would be a woman who a. Has a routine vaginal birth b. Has fetal shoulder dystocia c. Is delivering a preterm infant d. Has a history of rapid deliveries

ANS: B Feedback A Once routine for all vaginal deliveries, the perceived benefits of reducing pain and perineal tearing have not proven true. B An episiotomy is indicated in the situation where the shoulder of the fetus become lodged under the mother symphysis during birth. C A preterm infant is smaller and does not need as much space for delivery. D Rapid deliveries are not an indication for a mediolateral episiotomy.

14. For which patient should the oxytocin (Pitocin) infusion be discontinued immediately? a. A woman in active labor with contractions every 31 minutes lasting 60 seconds each b. A woman in transition with contractions every 2 minutes lasting 90 seconds each c. A woman in active labor with contractions every 2 to 3 minutes lasting 70 to 80 seconds each d. A woman in early labor with contractions every 5 minutes lasting 40 seconds each

ANS: B Feedback A Oxytocin may assist this woman's contractions to become closer and more efficient. B This woman's contraction pattern represents hyperstimulation, and inadequate resting time occurs between contractions to allow placental perfusion. C There is an appropriate resting period between this woman's contractions. D There is an appropriate resting period between this woman's contractions for her stage of labor.

3. The nurse caring for women in labor understands that childbirth pain is different from other types of pain in that it is a. More responsive to pharmacologic management b. Associated with a physiologic process c. Designed to make one withdraw from the stimulus d. Less intense

ANS: B Feedback A Pain management during labor may affect the course and length of labor. B Childbirth pain is part of a normal process, whereas other types of pain usually signify an injury or illness. C The pain with childbirth is a normal process; it is not caused by the type of injury when withdrawal from the stimuli is seen. D Childbirth pain is not less intense than other types of pain.

15. Which maternal factor may inhibit fetal descent and require further nursing interventions? a. Decreased peristalsis b. A full bladder c. Reduction in internal uterine size d. Rupture of membranes

ANS: B Feedback A Peristalsis does not influence fetal descent. B A full bladder may inhibit fetal descent because it occupies space in the pelvis needed by the fetal presenting part. C Contractions will reduce the internal uterine size in order to assist fetal descent. D Rupture of membranes will assist in the fetal descent.

21. The hips of a newborn are examined for developmental dysplasia. Which sign indicates an incomplete development of the acetabulum? a. Negative Ortolani's sign b. Thigh and gluteal creases are asymmetric c. Negative Barlow test d. Knee heights are equal

ANS: B Feedback A Positive Ortolani's sign yields a "clunking" sensation and indicates a dislocated femoral head moving into the acetabulum. B Asymmetric thigh and gluteal creases may indicate potential dislocation of the hip. C During a positive Barlow test, the examiner can feel the femoral head move out of acetabulum. D If the hip is dislocated, the knee on the affected side will be lower.

32. If a woman's fundus is soft 30 minutes after birth, the nurse's first response should be to a. Take the blood pressure. b. Massage the fundus. c. Notify the physician or nurse-midwife. d. Place the woman in Trendelenburg position.

ANS: B Feedback A The blood pressure is an important assessment to determine the extent of blood loss, but it is not the top priority. B The nurse's first response should be to massage the fundus to stimulate contraction of the uterus to compress open blood vessels at the placental site, limiting blood loss. C Notification should occur after all nursing measures have been attempted with no favorable results. D Trendelenburg position is contraindicated for this woman at this point. This position does not allow for appropriate vaginal drainage of lochia. The lochia remaining in the uterus would clot and produce further bleeding.

3. When positioning the Foley catheter before cesarean birth, the nurse knows that the catheter drainage tube and catheter bag should be a. Positioned on top of the patient's leg b. Placed near the head of the table c. Clamped during the cesarean section d. Positioned at the foot of the surgeon under the sterile drapes

ANS: B Feedback A The drain tube of the catheter should be positioned under her leg to promote drainage and to keep the catheter away from the operative area. B The anesthesia clinician must monitor urine output during the surgery. C Urinary output must be continuously monitored. An early sign of hypovolemia is a decreasing urinary output. D The surgeon might step on the drainage bag if the catheter was below the drapes, and no one could monitor the urine output.

16. When giving a narcotic to a laboring woman, the nurse should inject the medication at the beginning of a contraction so that a. Full benefit of the medication is received during that contraction. b. Less medication will be transferred to the fetus. c. The medication will be rapidly circulated. d. The maternal vital signs will not be adversely affected.

ANS: B Feedback A The full benefit will be received by the woman; however, it will decrease the amount reaching the fetus. B Injecting at the beginning of a contraction, when blood flow to the placenta is normally reduced, limits transfer to the fetus. C It will not increase the circulation of the medication. D It will not alter the vital signs any more than giving it at another time.

26. Which patient is most likely to experience pain during labor? a. Gravida 2 who has not attended childbirth preparation classes b. Gravida 2 who is anxious because her last labor was difficult c. Gravida 1 whose fetus is in a breech presentation d. Gravida 3 who is using Lamaze breathing techniques

ANS: B Feedback A The gravida 2 has previous experience and this will decrease anxiety. B Anxiety affects a woman's perception of pain. Tension during labor causes tightening of abdominal muscles, impeding contractions, and increasing pain by stimulation of nerve endings. C This woman will have more pain than if the infant is in vertex. Also, there is an increased likelihood that she will have a cesarean section and not go through labor. D The gravida 3 has previous experience and has prepared herself for the labor.

5. It is important for the nurse providing care during labor to be aware that pregnant women can usually tolerate the normal blood loss associated with childbirth because they have a. A higher hematocrit b. Increased blood volume c. A lower fibrinogen level d. Increased leukocytes

ANS: B Feedback A The hematocrit decreases with pregnancy due to the high fluid volume. B Women have a significant increase in blood volume during pregnancy. After delivery, the additional circulating volume is no longer necessary. C Fibrinogen levels increase with pregnancy. D Leukocyte levels increase during labor, but that is not the reason for the toleration of blood loss.

11. To teach and support the woman in labor, the nurse explains that the strongest part of a labor contraction is the a. Increment b. Acme c. Decrement d. Interval

ANS: B Feedback A The increment is the beginning of the contractions until it reaches the peak. B The acme is the peak or period of greatest strength during the middle of a contraction cycle. C The decrement occurs after the peak until the contraction ends. D The interval is the period between the end of the contraction and the beginning of the next.

10. A patient whose cervix is dilated to 5 cm is considered to be in which phase of labor? a. Latent phase b. Active phase c. Second stage d. Third stage

ANS: B Feedback A The latent phase is from the beginning of true labor until 3 cm of cervical dilation. B The active phase of labor is characterized by cervical dilation of 4 to 7 cm. C The second stage of labor begins when the cervix is completely dilated until the birth of the baby. D The third stage of labor is from the birth of the baby until the expulsion of the placenta.

2. After a forceps-assisted birth, the mother is observed to have continuous bright red lochia but a firm fundus. What other data indicates the presence of a potential vaginal wall hematoma? a. Mild, intermittent perineal pain b. Edema and discoloration of the labia and perineum c. Lack of an episiotomy d. Lack of pain in the perineal area

ANS: B Feedback A The pain with vaginal hematoma is severe and constant. B The nurse should monitor for edema and discoloration. Using a cold application to the labia and perineum reduces pain by numbing the area and limiting bruising and edema for the first 12 hours. C An episiotomy is performed as the fetal head distends the perineum. D The pain associated with vaginal hematoma is severe.

8. Cardiovascular changes that cause the foramen ovale to close at birth are a direct result of a. Increased pressure in the right atrium b. Increased pressure in the left atrium c. Decreased blood flow to the left ventricle d. Changes in the hepatic blood flow

ANS: B Feedback A The pressure in the right atrium decreases at birth. It is higher during fetal life. B With the increase in the blood flow to the left atrium from the lungs, the pressure is increased, and the foramen ovale is functionally closed. C Blood flow increases to the left ventricle after birth. D The hepatic blood flow changes, but that is not the reason for the closure of the foramen ovale.

3. A woman gave birth to a healthy 7-pound, 13-ounce infant girl. The nurse suggests that the woman place the infant to her breast within 15 minutes after birth. The nurse knows that breastfeeding is effective during the first 30 minutes after birth because this is the a. Transition period b. First period of reactivity c. Organizational stage d. Second period of reactivity

ANS: B Feedback A The transition period is the phase between intrauterine and extrauterine existence. B The first period of reactivity is the first phase of transition and lasts up to 30 minutes after birth. The infant is highly alert during this phase. C There is no such phase as the organizational stage. D The second period of reactivity occurs roughly between 4 and 8 hours after birth, after a period of prolonged sleep.

18. To improve placental blood flow immediately after the injection of an epidural anesthetic, the nurse should a. Turn the woman to the right side. b. Place a wedge under the woman's right hip. c. Give the woman oxygen. d. Decrease the intravenous infusion rate.

ANS: B Feedback A The woman needs to maintain the supine position for proper dispersal of the medication. However, placing a wedge under the hip will relieve compression of the vena cava. B Tilting the woman's pelvis to the left side relieves compression of the vena cava and compensates for a lower blood pressure without interfering with dispersal of the epidural medication. C Oxygen administration will not improve placental blood flow. D The intravenous infusion rate needs to be increased to prevent hypotension.

16. Which event indicates a complication of an external version? a. Maternal pulse rate of 100 bpm b. Fetal bradycardia persisting 10 minutes after the version c. Fetus returning to the original position d. Increased maternal anxiety after the version

ANS: B Feedback A There are few risks to the woman during an external version. B Fetal bradycardia after a version may indicate that the umbilical cord has become compressed, and the fetus is having hypoxia. C The fetus may return to the original position, but this is not a complication of the version. D Anxiety may occur before the version but should decrease after the procedure is completed.

6. It is important for the nurse to develop a realistic birth plan with the pregnant woman. The nurse can explain that a major advantage of nonpharmacologic pain management is that a. More complete pain relief is possible. b. No side effects or risks to the fetus are involved. c. The woman remains fully alert at all times. d. A more rapid labor is likely.

ANS: B Feedback A There is less pain relief with nonpharmacologic pain management during childbirth. B Because nonpharmacologic pain management does not include analgesics, adjunct drugs, or anesthesia, it is harmless to the mother and the fetus. C The woman's alertness is not altered by medication, but the increase in pain will decrease alertness. D Pain management may or may not alter the length of labor. At times when pain is decreased, the mother relaxes and labor progresses at a quicker pace.

1. The nurse sees a pattern on the fetal monitor that looks similar to early decelerations, but the deceleration begins near the acme of the contraction and continues well beyond the end of the contraction. Which nursing action indicates the proper evaluation of this situation? a. Continue to monitor these early decelerations, which occur as the fetal head is compressed during a contraction. b. This deceleration pattern is associated with uteroplacental insufficiency, so the nurse acts quickly to improve placental blood flow and fetal oxygen supply. c. This pattern reflects variable decelerations. No interventions are necessary at this time. d. Document this reassuring fetal heart rate pattern, but decrease the rate of the intravenous fluid.

ANS: B Feedback A These are late decelerations, not early; therefore interventions are necessary. B This is a description of a late deceleration. Oxygen should be given via snug facemask. Position the woman on her left side to increase placental blood flow. C Variable decelerations are caused by cord compression. A vaginal examination should be performed to identify this potential emergency. D This is not a reassuring pattern, so the intravenous rate should be increased to increase the mother's blood volume.

20. What is an appropriate response to a woman's comment that she is worried about having a cesarean birth? a. "Don't worry. Everything will be okay." b. "What are your feelings about having a cesarean birth?" c. "I know you're worried, but this is a routine procedure." d. "Patients commonly worry about surgery."

ANS: B Feedback A This answer is stating that the patient's feelings are not important. B Allowing the patient to express her feelings is the most appropriate nursing response. The nurse should never provide the patient with false reassurance or disregard her feelings. C This is belittling the patient's concerns and does not allow her to express her concerns. D This answer is close ended and belittling to the patient's feelings.

22. A woman who is gravida 3 para 2 enters the intrapartum unit. The most important nursing assessments are a. Contraction pattern, amount of discomfort, and pregnancy history b. Fetal heart rate, maternal vital signs, and the woman's nearness to birth c. Identification of ruptured membranes, the woman's gravida and para, and her support person d. Last food intake, when labor began, and cultural practices the couple desires

ANS: B Feedback A This is an important nursing assessment, but does not take priority if the birth is imminent. B All options describe relevant intrapartum nursing assessments, but the focus assessment has priority. If the maternal and fetal conditions are normal and birth is not imminent, other assessments can be performed in an unhurried manner. C This is an assessment that can occur later in the admission process if time permits. D This part of the assessment can occur later in the admission process if time permits.

27. A new mother asks, "Why are you doing a gestational age assessment on my baby?" The nurse's best response is a. "This must be done to meet insurance requirements." b. "It helps us identify infants who are at risk for any problems." c. "The gestational age determines how long the infant will be hospitalized." d. "It was ordered by your doctor."

ANS: B Feedback A This is not accurate information. B The nurse should provide the mother with accurate information about various procedures performed on the newborn. C Gestational age does not dictate hospital stays. Problems that occur due to gestational age may prolong the stay. D Assessing gestational age is a nursing assessment and does not have to be ordered.

6. To assess the duration of labor contractions, the nurse determines the time a. From the beginning of one contraction to the beginning of the next b. From the beginning to the end of each contraction c. Of the strongest intensity of each contraction d. Of uterine relaxation between two contractions

ANS: B Feedback A This is the frequency of the contractions. B Duration of labor contractions is the average length of contractions from beginning to end. C This is the strength or intensity of the contractions. D This is the interval of the contraction phase.

16. The process in which bilirubin is changed from a fat-soluble product to a water-soluble product is known as a. Enterohepatic circuit b. Conjugation of bilirubin c. Unconjugation of bilirubin d. Albumin binding

ANS: B Feedback A This is the route by which part of the bile produced by the liver enters the intestine, is reabsorbed by the liver, and then is recycled into the intestine. B Conjugation of bilirubin is the process of changing the bilirubin from a fat-soluble to a water-soluble product. C Unconjugated bilirubin is fat soluble. D Albumin binding is to attach something to a protein molecule.

28. Which intervention may be most helpful for the patient experiencing most of her labor pain in her back? a. Lying supine with head slightly elevated b. Having the support person apply firm pressure to the sacrum c. Sitting upright with the legs straight d. Having the nurse massage the upper back during a contraction

ANS: B Feedback A This would put more pressure onto the lower back area and decrease placental profusion. B Firm pressure against the sacrum may be helpful in relieving the discomfort associated with back labor. C This position would put more pressure onto the lower back area. D The massage should be in the lower back where the pain is located.

1. The nurse knows that a urinary catheter is added to the instrument table if a forceps-assisted birth is anticipated. What is the correct rationale for this intervention? a. Spontaneous release of urine might contaminate the sterile field. b. An empty bladder provides more room in the pelvis. c. A sterile urine specimen is needed preoperatively. d. A Foley catheter prevents the membranes from spontaneously rupturing.

ANS: B Feedback A Urine is sterile. B Catheterization provides room for the application of the forceps blades and limits bladder trauma. C A clean-catch urinalysis is usually sufficient for preoperative treatment. D The membranes must be ruptured and the cervix completely dilated for a forceps-assisted birth.

12. A maternal indication for the use of vacuum extraction is a. A wide pelvic outlet b. Maternal exhaustion c. A history of rapid deliveries d. Failure to progress past 0 station

ANS: B Feedback A With a wide pelvic outlet, vacuum extraction is not necessary. B With a mother who is exhausted will be unable to assist with the expulsion of the fetus. C With a rapid delivery, vacuum extraction is not necessary. D A station of 0 is too high for a vacuum extraction.

1. What are modes of heat loss in the newborn? (Choose all that apply.) a. Perspiration b. Convection c. Radiation d. Conduction e. Urination

ANS: B, C, D Feedback Correct Convection, radiation, evaporation, and conduction are the four modes of heat loss in the newborn. Incorrect Perspiration and urination are not modes of heat loss in newborns.

1. Labor is very stressful for the fetus. It is important for the intrapartum nurse to be knowledgeable regarding the mechanisms that regulate the fetal heart rate and keep the brain well oxygenated. When evaluating the patient's progress, the nurse knows that four of the five fetal factors that interact to regulate the heart rate are the a. Uterine activity b. Autonomic nervous system c. Baroreceptors d. Chemoreceptors e. Adrenal glands

ANS: B, C, D, E Feedback Correct The sympathetic and parasympathetic branches of the autonomic nervous system are balanced forces that regulate FHR. Sympathetic stimulation increases the heart rate, while parasympathetic responses, through stimulation of the vagus nerve, reduce the FHR and maintain variability. The baroreceptors stimulate the vagus nerve to slow the FHR and decrease the blood pressure. These are located in the carotid arch and major arteries. The chemoreceptors are cells that respond to changes in oxygen, carbon dioxide, and pH. They are found in the medulla oblongata and the aortic and carotid bodies. The adrenal medulla secretes epinephrine and norepinephrine in response to stress, causing an acceleration in FHR. Incorrect Hypertonic uterine activity can reduce the time available for exchange of oxygen and waste products; however, this is a maternal factor. The fifth fetal factor is the central nervous system. The fetal cerebral cortex causes the heart rate to increase during fetal movement and decrease when the fetus sleeps.

3. According to the NICHD Three-Tier System of Fetal Heart Rate Classification, Category III tracings include all FHR tracings not categorized as Category I or II. Which characteristics of the fetal heart belong in Category III? Select all that apply. a. Baseline rate of 110 to 160 bpm b. Tachycardia c. Absent baseline variability NOT accompanied by recurrent decelerations d. Variable decelerations with other characteristics such as shoulders or overshoots e. Absent baseline variability with recurrent variable decelerations f. Bradycardia

ANS: B, D, E, F Feedback Correct These characteristics are all considered non-reassuring or abnormal and belong in Category III. Incorrect A fetal heart rate of 110 to 160 bpm is considered normal and belongs in Category I. Absent baseline variability not accompanied by recurrent decelerations is a Category II characteristic.

35. When assessing the fetus using Leopold maneuvers, the nurse feels a round, firm, movable fetal part in the fundal portion of the uterus and a long, smooth surface in the mother's right side close to midline. What is the likely position of the fetus? a. ROA b. LSP c. RSA d. LOA

ANS: C Feedback Fetal position is denoted with a three-letter abbreviation. The first letter indicates the presenting part in either the right or left side of the maternal pelvis. The second letter indicates the anatomic presenting part of the fetus. The third letter stands for the location of the presenting part in relation to the anterior, posterior, or transverse portion of the maternal pelvis. Palpation of a round, firm fetal part in the fundal portion of the uterus would be the fetal head, indicating that the fetus is in a breech position with the sacrum as the presenting part in the maternal pelvis. Palpation of the fetal spine along the mother's right side denotes the location of the presenting part in the mother's pelvis. The ability to palpate the fetal spine indicates that the fetus is anteriorly positioned in the maternal pelvis. This fetus is positioned anteriorly in the right side of the maternal pelvis with the sacrum as the presenting part. RS/A/ is the correct three-letter abbreviation to indicate this fetal position.

21. Which patient at term should go to the hospital or birth center the soonest after labor begins? a. Gravida 2 para 1 who lives 10 minutes away b. Gravida 1 para 0 who lives 40 minutes away c. Gravida 3 para 2 whose longest previous labor was 4 hours d. Gravida 2 para 1 whose first labor lasted 16 hours

ANS: C Feedback A A gravida 2 is expected to have a longer labor than the gravida 3. The fact that she lives close to the hospital allows her to stay home for a longer period of time. B A gravida 1 is expected to have the longest labor. C Multiparous women usually have shorter labors than do nulliparous women. The woman described in option c is multiparous with a history of rapid labors, increasing the likelihood that her infant might be born in uncontrolled circumstances. D The gravida 2 is expected to have a longer labor than the gravida 3, especially since her first labor was 16 hours.

4. How does the available staff influence the selection of either continuous electronic or intermittent auscultation as the fetal-monitoring method? a. There must be a 1:1 nurse-to-patient ratio regardless of the method used. b. Staffing patterns do not influence fetal monitoring choices. c. Use of intermittent auscultation requires a lower nurse-to-patient ratio. d. More nurses are needed when electronic fetal monitoring is used because of increased medical interventions.

ANS: C Feedback A A one-to-one ratio is needed during the second stage of labor or if a high-risk condition exists, regardless of the monitoring method used. B Staffing patterns do plan a role in maintaining safe monitoring practice of the labor patient. C Intermittent auscultation is more staff-intensive. D Less nursing time is needed with electronic monitoring, giving the nurse more time for teaching and supporting the laboring woman.

19. Surgical, medical, or mechanical methods may be used for labor induction. Which technique is considered a mechanical method of induction? a. Amniotomy b. Intravenous Pitocin c. Transcervical catheter d. Vaginal insertion of prostaglandins

ANS: C Feedback A Amniotomy is a surgical method of augmentation and induction. B Intravenous Pitocin is a medical method of induction. C Placement of a balloon-tipped Foley catheter into the cervix is a mechanical method of induction. Other methods to expand and gradually dilate the cervix include Laminaria tents, Dilapan and Lamicel. D Insertion of prostaglandins is a medical method of induction.

26. A newborn who is large for gestational age (LGA) is _____ percentile for weight. a. Below the 90th b. Less than the 10th c. Greater than the 90th d. Between the 10th and 90th

ANS: C Feedback A An infant between the 10th and 90th percentiles is average for gestational age. B An infant in less than the 10th percentile is small for gestational age. C The LGA rating is based on weight and is defined as greater than the 90th percentile in weight. D This infant is considered average for gestational age.

28. Which nursing assessment indicates that a woman who is in second-stage labor is almost ready to give birth? a. The fetal head is felt at 0 station during vaginal examination. b. Bloody mucus discharge increases. c. The vulva bulges and encircles the fetal head. d. The membranes rupture during a contraction.

ANS: C Feedback A Birth of the head occurs when the station is +4. A 0 station indicates engagement. B Bloody show occurs throughout the labor process and is not an indication of an imminent birth. C A bulging vulva that encircles the fetal head describes crowning, which occurs shortly before birth. D Rupture of membranes can occur at any time during the labor process and does not indicate an imminent birth.

1. Childbirth preparation can be considered successful if the outcome is described as a. Labor was pain-free. b. Birth experiences of friends and families were discredited. c. The woman rehearsed labor and practiced skills to master pain. d. Only nonpharmacologic methods for pain control were used.

ANS: C Feedback A Childbirth preparation does not guarantee a pain-free labor. A woman should be prepared for pain and anesthesia/analgesia realistically. B Friends and families can be an important source of support if they convey realistic information about labor pain. C Preparation allows the woman to rehearse for labor and to learn new skills to cope with the pain of labor and the expected behavioral changes. D Women will not always achieve their desired level of pain control by using nonpharmacologic methods alone.

8. Which factor ensures that the smallest anterior-posterior diameter of the fetal head enters the pelvis? a. Descent b. Engagement c. Flexion d. Station

ANS: C Feedback A Descent is the moving of the fetus through the birth canal. B Engagement occurs when the largest diameter of the fetal presenting part has passed the pelvic inlet. C Flexion of the fetal head allows the smallest head diameters pass through the pelvis. D The station is the relationship of the fetal presenting part to the level of the ischial spines.

15. The laboring woman who imagines her body opening to let the baby out is using a mental technique called a. Dissociation b. Effleurage c. Imagery d. Distraction

ANS: C Feedback A Dissociation helps the woman learn to relax all muscles except those that are working. B Effleurage is self-massage. C Imagery is a technique of visualizing images that will assist the woman in coping with labor. D Distraction can be used in the early latent phase by having the woman involved in another activity.

12. Which statement is true about the physiologic effects of pain in labor? a. It usually results in a more rapid labor. b. It is considered to be a normal occurrence. c. It may result in decreased placental perfusion. d. It has no effect on the outcome of labor.

ANS: C Feedback A Excessive pain may prolong the labor due to increase anxiety in the woman. B Pain is considered normal for labor. However, excessive pain may be an indication of other problems and must be assessed. C When experiencing excessive pain, the woman may react with a stress response that diverts blood flow from the uterus and the fetus. D It may affect the outcome of the labor depending on the cause and the effect on the woman.

29. What characteristic shows the greatest gestational maturity? a. Few rugae on the scrotum and testes high in the scrotum b. Infant's arms and legs extended c. Some peeling and cracking of the skin d. The arm can be positioned with the elbow beyond the midline of the chest

ANS: C Feedback A Few rugae on the scrotum show a younger age in the newborn. B Extended arms and legs is a sign of preterm infants. C Peeling, cracking, dryness, and a few visible veins in the skin are signs of maturity in the newborn. D This result of the scarf sign shows a younger newborn.

19. In which infant behavioral state is bonding most likely to occur? a. Drowsy b. Active alert c. Quiet alert d. Crying

ANS: C Feedback A In the drowsy state the eyes may remain closed. If open they are unfocused. The infant is not interested in the environment at this time. B In the active alert state infants are often fussy, restless, and not focused. C In the quiet alert state, the infant is interested in his or her surroundings and will often gaze at the mother or father or both. D During the crying state the infant does not respond to stimulation and cannot focus on parents.

10. The nurse caring for a woman in labor understands that the primary risk associated with an amniotomy is a. Maternal infection b. Maternal hemorrhage c. Prolapse of the umbilical cord d. Separation of the placenta

ANS: C Feedback A Infection is a risk of amniotomy, but not the primary concern. B Maternal hemorrhage is not associated with amniotomy. C When the membranes are ruptured, the umbilical cord may come downward with the flow of amniotic fluid and become trapped in front of the presenting part. D This may occur if the uterus is overdistended before the amniotomy, but it is not the major concern.

9. What results from the adaptation of the fetus to the size and shape of the pelvis? a. Lightening b. Lie c. Molding d. Presentation

ANS: C Feedback A Lightening is the descent of the fetus toward the pelvic inlet before labor. B Lie is the relationship of the long axis of the fetus to the long axis of the mother. C The sutures and fontanels allow the bones of the fetal head to move slightly, changing the shape of the fetal head so it can adapt to the size and shape of the pelvis. D Presentation is the fetal part that first enters the pelvic inlet.

22. To monitor for potential hemorrhage in the woman who has just had a cesarean birth, the recovery room nurse should a. Maintain an intravenous infusion at 100 mL/hr. b. Assess the abdominal dressings for drainage. c. Assess the uterus for firmness every 15 minutes. d. Monitor her urinary output.

ANS: C Feedback A Maintaining proper fluid balance will not control hemorrhage. B This is an important assessment, but hemorrhage will first be noted vaginally. C Maintaining contraction of the uterus is important in controlling bleeding from the placental site. D This is an important assessment to prevent future hemorrhaging from occurring, but it is not the first priority assessment in the recovery room.

14. A meconium stool can be differentiated from a transitional stool in the newborn because the meconium stool is a. Seen at age 3 days b. The residue of a milk curd c. Passed in the first 12 hours of life d. Lighter in color and looser in consistency

ANS: C Feedback A Meconium stool is the first stool of the newborn. B Meconium stool is made up of matter in the intestines during intrauterine life. C Meconium stool is usually passed in the first 12 hours of life and 99% of newborns have their first stool within 48 hours. If meconium is not passed by 48 hours, obstruction is suspected. D Meconium is dark in color and sticky.

24. A maculopapular rash with a red base and a small white papule in the center is a. Milia b. Mongolian spots c. Erythema toxicum d. Cafe-au-lait spots

ANS: C Feedback A Milia are minute epidermal cysts on the face of the newborn. B Mongolian spots are bluish-black discolorations found on dark-skinned newborns, usually on the sacrum. C This is a description of erythema toxicum, a normal rash in the newborn. D These spots are pale tan (the color of coffee with milk) macules. Occasional spots occur normally in newborns.

15. The priority nursing care associated with an oxytocin (Pitocin) infusion is a. Measuring urinary output b. Increasing infusion rate every 30 minutes c. Monitoring uterine response d. Evaluating cervical dilation

ANS: C Feedback A Monitoring urinary output is important with Pitocin, but not the top priority. B The infusion rate may be increased, but only after proper assessment that it is appropriate. C Because of the risk of hyperstimulation, which could result in decreased placental perfusion and uterine rupture, the nurse's priority intervention is monitoring uterine response. D Monitoring labor progression is important, but not the top priority.

5. A first-time dad is concerned that his 3-day-old daughter's skin looks "yellow." In the nurse's explanation of physiologic jaundice, what fact should be included? a. Physiologic jaundice occurs during the first 24 hours of life. b. Physiologic jaundice is caused by blood incompatibilities between the mother and infant blood types. c. The bilirubin levels of physiologic jaundice peak between the second and fourth days of life. d. This condition is also known as "breast milk jaundice."

ANS: C Feedback A Pathologic jaundice occurs during the first 24 hours of life. B Pathologic jaundice is caused by blood incompatibilities, causing excessive destruction of erythrocytes, and must be investigated. C Physiologic jaundice becomes visible when the serum bilirubin reaches a level of 5 mg/dL or greater, which occurs when the baby is approximately 3 days old. This finding is within normal limits for the newborn. D Breast milk jaundice occurs in one third of breastfed infants at 2 weeks and is caused by an insufficient intake of fluids.

29. During labor, a vaginal examination should be performed only when necessary because of the risk of a. Fetal injury b. Discomfort c. Infection d. Perineal trauma

ANS: C Feedback A Properly performed vaginal examinations should not cause fetal injury. B Vaginal examinations may be uncomfortable for some women in labor, but that is not the main reason for limiting them. C Vaginal examinations increase the risk of infection by carrying vaginal microorganisms upward toward the uterus. D A properly performed vaginal examination should not cause perineal trauma.

14. When a nonreassuring pattern of the fetal heart rate is noted and the mother is lying on her left side, what nursing action is indicated? a. Lower the head of the bed. b. Place the mother in a Trendelenburg position. c. Change her position to the right side. d. Place a wedge under the left hip.

ANS: C Feedback A Repositioning to the opposite side is the first intervention. If unsuccessful with improving the FHR pattern, further changes in position can be attempted. B The Trendelenburg position is not appropriate for early interventions. If unsuccessful with improving the FHR pattern with other types of position changes, Trendelenburg may be the choice. C Repositioning on the opposite side may relieve compression on the umbilical cord and improve blood flow to the placenta. D The woman is already on her left side, so a wedge on that side is not an appropriate choice.

33. The nurse thoroughly dries the infant immediately after birth primarily to a. Stimulate crying and lung expansion. b. Remove maternal blood from the skin surface. c. Reduce heat loss from evaporation. d. Increase blood supply to the hands and feet.

ANS: C Feedback A Rubbing the infant does stimulate crying, but it is not the main reason for drying the infant. B Drying the infant after birth does not remove all of the maternal blood. C Infants are wet with amniotic fluid and blood at birth, which accelerates evaporative heat loss. D The main purpose of drying the infant is to prevent heat loss.

11. What is a result of hypothermia in the newborn? a. Shivering to generate heat b. Decreased oxygen demands c. Increased glucose demands d. Decreased metabolic rate

ANS: C Feedback A Shivering is not an effective method of heat production for newborns. B Oxygen demands increase with hypothermia. C In hypothermia, the basal metabolic rate (BMR) is increased in an attempt to compensate, thus requiring more glucose. D The metabolic rate increases with hypothermia.

6. To provide competent newborn care, the nurse understands that respirations are initiated at birth as a result of a. An increase in the PO2 and a decrease in PCO2 b. The continued functioning of the foramen ovale c. Chemical, thermal, sensory, and mechanical factors d. Drying off the infant

ANS: C Feedback A The PO2 decreases at birth and the PCO2 increases. B The foramen ovale closes at birth. C A variety of these factors are responsible for initiation of respirations. D Tactile stimuli aid in initiating respirations, but are not the main cause.

31. At 1 minute after birth, the nurse assesses the newborn to assign an Apgar score. The apical heart rate is 110 bpm, and the infant is crying vigorously with the limbs flexed. The infant's trunk is pink, but the hands and feet are blue. What is the Apgar score for this infant? a. 7 b. 8 c. 9 d. 10

ANS: C Feedback A The baby received 2 points for each of the categories except color. Since the infant's hands and feet were blue this category is given a grade of 1. B The baby received 2 points for each of the categories except color. Since the infant's hands and feet were blue this category is given a grade of 1. C The Apgar score is 9 because 1 point is deducted from the total score of 10 for the infant's blue hands and feet. D The infant had 1 point deducted because of the blue color of the hands and feet.

4. To provide safe care for the woman, the nurse understands that which condition is a contraindication for an amniotomy? a. Dilation less than 3 cm b. Cephalic presentation c. -2 station d. Right occiput posterior position

ANS: C Feedback A The dilation must be enough to determine labor. B The presenting part should be cephalic. Amniotomy is deferred if the presenting part is higher in the pelvis. C A prolapsed cord can occur if the membranes artificially rupture when the presenting part is not engaged. D This indicates a cephalic presentation, which is appropriate for an amniotomy.

3. Which method of assessing the fetal heart rate requires the use of a gel? a. Fetoscope b. Tocodynamometer c. Doppler d. Scalp electrode

ANS: C Feedback A The fetoscope does not require the use of gel because ultrasonic transmission is not used. B Tocodynamometer does not require the use of gel. This device monitors uterine contractions. C Doppler is the only listed method involving ultrasonic transmission of fetal heart rates; it requires use of a gel. D The scalp electrode is attached to the fetal scalp; gel is not necessary.

17. Which statement is correct regarding the fluid balance in a newborn versus that in an adult? a. The infant has a smaller percentage of surface area to body mass. b. The infant has a smaller percentage of water to body mass. c. The infant has a greater percentage of insensible water loss. d. The infant has a 50% more effective glomerular filtration rate.

ANS: C Feedback A The infant's surface area is large compared to an adult's. B Infants have a larger percentage of water to body mass. C Insensible water loss is greater in the infant due to the newborn's large body surface area and rapid respiratory rate. D The filtration rate is less than in adults; the kidneys are immature in a newborn.

1. The maternity nurse understands that as the uterus contracts during labor, maternal-fetal exchange of oxygen and waste products a. Continues except when placental functions are reduced b. Increases as blood pressure decreases c. Diminishes as the spiral arteries are compressed d. Is not significantly affected

ANS: C Feedback A The maternal blood supply to the placenta gradually stops with contractions. B The exchange of oxygen and waste products decreases. C During labor contractions, the maternal blood supply to the placenta gradually stops as the spiral arteries supplying the intervillous space are compressed by the contracting uterine muscle. D The exchange of oxygen and waste products is affected by contractions.

11. The most important nursing intervention after the injection of epidural anesthesia is monitoring a. Urinary output b. Contractions c. Maternal blood pressure d. Intravenous infusion rate

ANS: C Feedback A The mother is prone to bladder distention and this needs to be monitored. However, this does not become a problem until 1 to 3 hours later. B Continuing monitoring of the contractions is important, because they may be altered by the epidural. However, this is not the most important nursing intervention after the epidural is administered. C Epidural anesthesia may produce maternal hypotension due to vasodilation. D The intravenous infusion needs to be maintained before, during, and after the epidural. However, it is not the most important nursing intervention at this point.

10. While assessing the newborn, the nurse should be aware that the average expected apical pulse range of a full-term, quiet, alert newborn is _____ beats/min. a. 80 to 100 b. 100 to 120 c. 120 to 160 d. 150 to 180

ANS: C Feedback A The newborn's heart rate may be about 85 to 100 beats/min while sleeping. B The infant's heart rate typically is a bit higher when alert but quiet. C The average infant heart rate while awake is 120 to 160 beats/min. D A heart rate of 150 to 180 beats/min is typical when the infant cries.

37. With regard to the newborn's developing cardiovascular system, nurses should be aware that a. The heart rate of a crying infant may rise to 120 beats/min. b. Heart murmurs heard after the first few hours are cause for concern. c. The point of maximal impulse (PMI) often is visible on the chest wall. d. Persistent bradycardia may indicate respiratory distress syndrome (RDS).

ANS: C Feedback A The normal heart rate for infants who are not sleeping is 120 to 160 beats/min. However, a crying infant temporarily could have a heart rate of 180 beats/min. B Heart murmurs during the first few days of life have no pathologic significance; an irregular heart rate past the first few hours should be evaluated further. C The newborn's thin chest wall often allows the PMI to be seen. D Persistent tachycardia may indicate RDS; bradycardia may be a sign of congenital heart blockage.

17. Which nursing action is correct when initiating electronic fetal monitoring? a. Lubricate the tocotransducer with an ultrasound gel. b. Inform the patient that she should remain in the semi-Fowler's position. c. Securely apply the tocotransducer with a strap or belt. d. Determine the position of the fetus before attaching the electrode to the maternal abdomen.

ANS: C Feedback A The tocotransducer does not need gel to operate appropriately. B The patient should be encouraged to move around during labor. C The tocotransducer should fit snugly on the abdomen to monitor uterine activity accurately. D The tocotransducer should be placed at the fundal area of the uterus.

9. The nurse-midwife is concerned that a woman's uterine activity is too intense and that her obesity is preventing accurate assessment of the actual intrauterine pressure. On the basis of this information, the nurse should obtain a(n) a. Tocotransducer b. Scalp electrode c. Intrauterine pressure catheter d. Doppler transducer

ANS: C Feedback A The tocotransducer measures the uterine pressure externally; this not be accurate with obesity. B A scalp electrode measurers the fetal heart rate (FHR). C An intrauterine pressure catheter can measure actual intrauterine pressure. D A Doppler auscultates the FHR.

13. Which woman will most likely have increased anxiety and tension during her labor? a. Gravida 1 who did not attend prepared childbirth classes b. Gravida 2 who refused any medication c. Gravida 2 who delivered a stillborn baby last year d. Gravida 3 who has two children younger than 3 years

ANS: C Feedback A The woman is not prepared for labor and will have increased anxiety during labor. However, the woman with a poor previous outcome is more likely to experience more anxiety. B A gravida 2 has previous experience and can anticipate what to expect. By refusing any medication, she is taking control over her situation and will have less anxiety. C If a previous pregnancy had a poor outcome, the woman will probably be more anxious during labor and delivery. D This gravida 3 has previous experience and is aware of what to expect.

8. The greatest risk to the newborn after an elective cesarean birth is a. Trauma due to manipulation during delivery b. Tachypnea due to maternal anesthesia c. Prematurity due to miscalculation of gestation d. Tachycardia due to maternal narcotics

ANS: C Feedback A There is reduced trauma with a cesarean birth. B Maternal anesthesia may cause respiratory distress. C Regardless of the many criteria used to determine gestational age, inadvertent preterm birth still occurs. D Maternal narcotics may cause respiratory distress.

20. Heat loss by convection occurs when a newborn is a. Placed on a cold circumcision board b. Given a bath c. Placed in a drafty area of the room d. Wrapped in cool blankets

ANS: C Feedback A This is conduction. B This is evaporation. C Convection occurs when infants are exposed to cold air currents. D This is conduction.

15. The nurse notes a pattern of late decelerations on the fetal monitor. The most appropriate action is to a. Continue observation of this reassuring pattern. b. Notify the physician or nurse-midwife. c. Give the woman oxygen by face mask. d. Place the woman in a Trendelenburg position.

ANS: C Feedback A This is not a reassuring pattern; interventions are needed. B Nursing interventions should be initiated before notifying the health care provider. C Late decelerations are associated with reduced placental perfusion. Giving the laboring woman oxygen increases the oxygen saturation in her blood, making more oxygen available to the fetus. D The Trendelenburg position will not increase the placental perfusion.

13. In administering vitamin K to the infant shortly after birth, the nurse understands that vitamin K is a. Important in the production of red blood cells b. Necessary in the production of platelets c. Not initially synthesized because of a sterile bowel at birth d. Responsible for the breakdown of bilirubin and prevention of jaundice

ANS: C Feedback A Vitamin K is important for blood clotting. B The platelet count in term newborns is near adult levels. Vitamin K is necessary to activate prothrombin and other clotting factors. C The bowel is initially sterile in the newborn, and vitamin K cannot be synthesized until food is introduced into the bowel. D Vitamin K is necessary to activate the clotting factors.

2. The nurse is caring for a laboring patient who develops a fever after she has had her epidural initiated. Fever with no apparent infection may occur in women who have epidural analgesia. The neonate's temperature may be elevated as well, possibly leading to unnecessary treatment for neonatal sepsis. Possible explanations for epidural associated fever without infection include (select all that apply) a. Bladder distention b. Maternal hypotension c. Reduction of heat dissipation d. Vasodilation e. Shivering

ANS: C, D, E Feedback Correct Heat dissipation is reduced as a result of decreased hyperventilation, sweating, and activity after the onset of pain relief. Vasodilation redistributes heat from the core to the periphery of the body, where it is lost to the environment. Shivering often occurs with sympathetic blockade accompanied by a dissociation between warm and cold sensations. In essence, the body believes that the temperature is lower than it actually is and turns up the "thermostat." Incorrect Bladder distention is an anticipated effect of having an epidural. A woman's bladder fills quickly because of the large quantity of IV solution, yet her sensation to void is reduced. Maternal hypotension is an expected side effect of epidural initiation.

2. The baseline fetal heart rate (FHR) is the average rate during a 10-minute segment. Changes in FHR are categorized as periodic or episodic. These patterns include both accelerations and decelerations. The labor nurse is evaluating the patient's most recent 10-minute segment on the monitor strip and notes a late deceleration. This is likely to be caused by which physiologic alteration? Select all that apply. a. Spontaneous fetal movement b. Compression of the fetal head c. Placental abruption d. Cord around the baby's neck e. Maternal supine hypotension

ANS: C, E Feedback Correct Late decelerations are almost always caused by uteroplacental insufficiency. Insufficiency is caused by: uterine tachysystole, maternal hypotension, epidural or spinal anesthesia, IUGR, intraamniotic infection, or placental abruption. Incorrect Spontaneous fetal movement, vaginal examination, fetal scalp stimulation, fetal reaction to external sounds, uterine contractions, fundal pressure and abdominal palpation are all likely to cause accelerations of the FHR. Early decelerations are most often the result of fetal head compression and may be caused by uterine contractions, fundal pressure, vaginal examination and placement of an internal electrode. A variable deceleration is likely caused by umbilical cord compression. This may happen when the cord is around the baby's neck, arm, leg or other body part, a short cord, a knot in the cord or a prolapsed cord.

18. Which assessment finding could indicate hemorrhage in the postpartum patient? a. Firm fundus at the midline b. Saturation of two perineal pads in 4 hours c. Elevated blood pressure d. Elevated pulse rate

ANS: D Feedback A A firm fundus indicates that the uterus is contracting and compressing the open blood vessels at the placental site. B Saturation of one pad within the first hour is the maximum normal amount of lochial flow. Two pads within 4 hours is within normal limits. C If the blood volume were diminishing, the blood pressure would decrease. D An increasing pulse rate is an early sign of excessive blood loss.

18. When preparing a woman for a cesarean birth, the nurse's care should include a. Injection of narcotic preoperative medications b. Full perineal shave preparation c. Straight catheterization to empty the bladder d. Administration of an oral antacid

ANS: D Feedback A A narcotic at this point would put the fetus at high risk for respiratory distress. B Perineal preparation is not necessary for a cesarean section. Some agencies will do an abdominal prep just before the surgery. C The catheterization should be indwelling in order to keep the bladder small during the surgery. D General anesthesia may be needed unexpectedly for cesarean birth. An oral antacid neutralizes gastric acid and reduces potential lung injury if the woman vomits and aspirates gastric contents during anesthesia.

26. What finding should the nurse recognize as being associated with fetal compromise? a. Active fetal movements b. Contractions lasting 90 seconds c. FHR in the 140s d. Meconium-stained amniotic fluid

ANS: D Feedback A Active fetal movement is an expected occurrence. B The fetus should be able to tolerate contractions lasting 90 seconds if the resting phase is sufficient to allow for a return of adequate blood flow. C Expected FHR range is from 120 to 160. D When fetal oxygen is compromised, relaxation of the rectal sphincter allows passage of meconium into the amniotic fluid.

5. When providing labor support, the nurse knows that which fetal position might cause the laboring woman more back discomfort? a. Right occiput anterior b. Left occiput anterior c. Right occiput transverse d. Left occiput posterior

ANS: D Feedback A Back labor is seen mostly when the fetus is in the posterior position. B Back labor is seen mostly when the fetus is in the posterior position. C Back labor is seen mostly when the fetus is in the posterior position. D In the left occiput posterior position, each contraction pushes the fetal head against the mother's sacrum, which results in intense back discomfort.

12. The infant with the lowest risk of developing high levels of bilirubin is the one who a. Was bruised during a difficult delivery b. Developed a cephalhematoma c. Uses brown fat to maintain temperature d. Breastfeeds during the first hour of life

ANS: D Feedback A Bruising will release more bilirubin into the system. B Cephalhematomas will release bilirubin into the system as the red blood cells die off. C Brown fat is normally used to produce heat in the newborn. D The infant who is fed early will be less likely to retain meconium and reabsorb bilirubin from the intestines back into the circulation.

21. A nurse might be called on to stimulate the fetal scalp a. As part of fetal scalp blood sampling b. In response to tocolysis c. In preparation for fetal oxygen saturation monitoring d. To elicit an acceleration in the FHR

ANS: D Feedback A Fetal scalp blood sampling involves swabbing the scalp with disinfectant before a sample is collected. The nurse would stimulate the fetal scalp to elicit an acceleration of the FHR. B Tocolysis is relaxation of the uterus. The nurse would stimulate the fetal scalp to elicit an acceleration of the FHR. C Fetal oxygen saturation monitoring involves the insertion of a sensor. The nurse would stimulate the fetal scalp to elicit an acceleration of the FHR. D The scalp can be stimulated using digital pressure during a vaginal examination.

6. The standard of care for obstetrics dictates that an internal version might be used to manipulate the a. Fetus from a breech to a cephalic presentation before labor begins b. Fetus from a transverse lie to a longitudinal lie before cesarean birth c. Second twin from an oblique lie to a transverse lie before labor begins d. Second twin from a transverse lie to a breech presentation during vaginal birth

ANS: D Feedback A For internal version to occur, the cervix needs to be completely dilated. B For internal version to occur, the cervix needs to be dilated. C Internal version is done to turn the second twin after the first twin is born. D Internal version is used only during vaginal birth to manipulate the second twin into a presentation that allows it to be born vaginally.

16. Increasing the infusion rate of nonadditive intravenous fluids can increase fetal oxygenation primarily by a. Maintaining normal maternal temperature b. Preventing normal maternal hypoglycemia c. Increasing the oxygen-carrying capacity of the maternal blood d. Expanding maternal blood volume

ANS: D Feedback A Increasing fluid volume may alter the maternal temperature only if she is dehydrated. B Most intravenous fluids for laboring women are isotonic and do not add extra glucose. C Oxygen-carrying capacity is increased by adding more red blood cells. D Filling the mother's vascular system makes more blood available to perfuse the placenta and may correct hypotension.

31. An African-American woman noticed some bruises on her newborn girl's buttocks. She asks the nurse who spanked her daughter. The nurse explains that these marks are called a. Lanugo b. Vascular nevi c. Nevus flammeus d. Mongolian spots

ANS: D Feedback A Lanugo is the fine, downy hair seen on a term newborn. B A vascular nevus, commonly called a strawberry mark, is a type of capillary hemangioma. C A nevus flammeus, commonly called a port-wine stain, is most frequently found on the face. D A Mongolian spot is a bluish black area of pigmentation that may appear over any part of the exterior surface of the body. It is more commonly noted on the back and buttocks and most frequently is seen on infants whose ethnic origins are Mediterranean, Latin American, Asian, or African.

9. The nurse practicing in a labor setting knows that the woman most at risk for a uterine rupture is a gravida a. 3 who has had two low-segment transverse cesarean births b. 2 who had a low-segment vertical incision for delivery of a 10-pound infant c. 5 who had two vaginal births and two cesarean births d. 4 who has had all cesarean births

ANS: D Feedback A Low-segment transverse cesarean scars do not predispose her to uterine rupture. B Low-segment incisions do not raise the risk of uterine ruptures. C This woman is not a high-risk candidate. D The risk of uterine rupture increases as the number of prior uterine incisions increases. More than 2 previous cesarean births places the woman at increased risk for uterine rupture.

9. A woman received 50 mcg of Fentanyl intravenously 1 hour before delivery. What drug should the nurse have readily available? a. Promethazine (Phenergan) b. Nalbuphine (Nubain) c. Butorphanol (Stadol) d. Naloxone (Narcan)

ANS: D Feedback A Phenergan is normally given for nausea. B Nubain is an analgesic given to women in labor. C Stadol is an analgesic given to women in labor. D Naloxone (Narcan) reverses narcotic-induced respiratory depression, which may occur with administration of narcotic analgesia.

14. Which method of pain management is safest for a gravida 3 para 2 admitted at 8 cm cervical dilation? a. Epidural anesthesia b. Narcotics c. Spinal block d. Breathing and relaxation techniques

ANS: D Feedback A Probably not enough time remains to administer epidural anesthesia. B A narcotic given at this time may reach its peak about the time of birth and result in respiratory depression in the newborn. C Probably not enough time remains to administer spinal anesthesia. D Nonpharmacologic methods of pain management may be the best option for a woman in advanced labor.

20. One of the greatest risks to the mother during administration of general anesthesia is a. Respiratory depression b. Uterine relaxation c. Inadequate muscle relaxation d. Aspiration of stomach contents

ANS: D Feedback A Respirations can be altered during general anesthesia, and the anesthesiologist will take precautions to maintain proper oxygenation. B Uterine relaxation can occur with some anesthesia, but this can be monitored and prevented. C Inadequate muscle relaxation can be altered. This is not the greatest risk for the mother. D Aspiration of acidic gastric contents and possible airway obstruction is a potentially fatal complication of general anesthesia.

17. Which comfort measure should the nurse use to assist the laboring woman to relax? a. Keep the room lights lit so that the patient and her coach can see everything. b. Offer warm, wet cloths to use on the patient's face and neck. c. Palpate her filling bladder every 15 minutes. d. Recommend frequent position changes.

ANS: D Feedback A Soft, indirect lighting is more soothing than irritating bright lights. B Women in labor become hot and perspire. Cool cloths are much better C A full bladder intensifies labor pain. The bladder should be emptied every 2 hours. D Frequent maternal position changes reduce the discomfort from constant pressure and promote fetal descent.

4. To adequately care for patients, the nurse understands that labor contractions facilitate cervical dilation by a. Contracting the lower uterine segment b. Enlarging the internal size of the uterus c. Promoting blood flow to the cervix d. Pulling the cervix over the fetus and amniotic sac

ANS: D Feedback A The contractions are stronger at the fundus. B The internal size becomes smaller with the contractions; this helps to push the fetus down. C Blood flow decreases to the uterus during a contraction. D Effective uterine contractions pull the cervix upward at the same time that the fetus and amniotic sac are pushed downward.

17. The method of anesthesia in labor considered the safest for the fetus is the a. Pudendal block b. Epidural block c. Spinal (subarachnoid) block d. Local infiltration

ANS: D Feedback A The fetus can be affected by maternal side effects. B The fetus can be affected by maternal hypotension. C The fetus can be affected by maternal side effects. D Local infiltration of the perineum rarely has any adverse effects on either the mother or the fetus.

15. When the newborn infant is fed, the most likely cause of regurgitation is a. Placing the infant in a prone position after a feeding b. The gastrocolic reflex c. An underdeveloped pyloric sphincter d. A relaxed cardiac sphincter

ANS: D Feedback A The infant should be placed in a supine position. B The gastrocolic reflex increases intestinal peristalsis after the stomach fills. C The pyloric sphincter goes from the stomach to the intestines. D The underlying cause of newborn regurgitation is a relaxed cardiac sphincter.

29. Which technique could the support person use when the laboring woman appears to be losing control? a. Have the nurse take over the role of support. b. Tell the woman that she is causing stress to her baby and herself. c. Wait for the contraction to end and discuss the problem with her. d. Make eye contact with the woman and breathe along with her.

ANS: D Feedback A The woman already has a trusting relationship with the support person so they should stay in that position if possible. B Imagery involves the woman creating a relaxing mental scene and dissociating herself from the painful aspects of labor. C Mental stimulation occupies the woman's mind and competes with pain stimuli. D Making eye contact and breathing along with the laboring woman to help pace her breathing will assist her in remaining calm.

37. A pregnant woman is at 38 weeks of gestation. She wants to know if any signs indicate "labor is getting closer to starting." The nurse informs the woman that which of the following is a sign that labor may begin soon? a. Weight gain of 1.5 to 2 kg (3 to 4 lb) b. Increase in fundal height c. Urinary retention d. Surge of energy

ANS: D Feedback A The woman may lose 0.5 to 1.5 kg, the result of water loss caused by electrolyte shifts, which in turn are caused by changes in the estrogen and progesterone levels. B When the fetus descends into the true pelvis (called lightening), the fundal height may decrease. C Urinary frequency may return before labor. D Women speak of having a burst of energy before labor.

27. Which type of cutaneous stimulation involves massage of the abdomen? a. Thermal stimulation b. Imagery c. Mental stimulation d. Effleurage

ANS: D Feedback A Thermal stimulation is the use of warmth to provide comfort, such as showers and baths. B Imagery involves the woman creating a relaxing mental scene and dissociating herself from the painful aspects of labor. C Mental stimulation occupies the woman's mind and competes with pain stimuli. D Effleurage is massage usually performed on the abdomen during contractions.

23. A primigravida at 39 weeks of gestation is observed for 2 hours in the intrapartum unit. The fetal heart rate has been normal. Contractions are 5 to 9 minutes apart, 20 to 30 seconds in duration, and of mild intensity. Cervical dilation is 1 to 2 cm and uneffaced (unchanged from admission). Membranes are intact. The nurse should expect the woman to be a. Admitted and prepared for a cesarean birth b. Admitted for extended observation c. Discharged home with a sedative d. Discharged home to await the onset of true labor

ANS: D Feedback A These are all indications of false labor without fetal distress. There is no indication that a cesarean birth is indicated. B These are all indications of false labor; there is no indication that further assessment or observations are indicated. C The patient will probably be discharged, but there is no indication that a sedative is needed. D The situation describes a woman with normal assessments who is probably in false labor and will probably not deliver rapidly once true labor begins.

1. A nursing student is helping the nursery nurses with morning vital signs. A baby born 10 hours ago via cesarean section is found to have moist lung sounds. What is the best interpretation of these data? a. The nurse should notify the pediatrician stat for this emergency situation. b. The neonate must have aspirated surfactant. c. If this baby was born vaginally, it could indicate a pneumothorax. d. The lungs of a baby delivered by cesarean section may sound moist for 24 hours after birth.

ANS: D Feedback A This is a common condition for infants delivered by cesarean section. B Surfactant is produced by the lungs, so aspiration is not a concern. C It is common to have some fluid left in the lungs; this will be absorbed within a few hours. D The condition will resolve itself within a few hours. For this common condition of newborns, surfactant acts to keep the expanded alveoli partially open between respirations. In vaginal births, absorption of remaining lung fluid is accelerated by the process of labor and delivery. Remaining lung fluid will move into interstitial spaces and be absorbed by the circulatory and lymphatic systems.

11. When the deceleration pattern of the fetal heart rate mirrors the uterine contraction, which nursing action is indicated? a. Administer oxygen by nasal cannula. b. Reposition the woman. c. Apply a fetal scalp electrode. d. Record this reassuring pattern.

ANS: D Feedback A This is an early deceleration; it is reassuring. B This is an early deceleration; it is reassuring. C This is an early deceleration; it is reassuring. D The periodic pattern described is early deceleration that is not associated with fetal compromise and requires no intervention.

30. A sign of illness in the newborn is a. More than two soft stools per day b. Regurgitating a small amount of feeding c. A yellow scaly lesion on the scalp d. An axillary temperature greater than 37.5° C

ANS: D Feedback A This is an expected finding in the newborn. B This is an expected finding in the newborn. C This is a sign of cradle cap or seborrhea capitis. D Infants commonly respond to a variety of illnesses with an elevation in temperature. The normal range for an axillary temperature in the newborn is 36.5° to 37.3° C.

13. The priority nursing intervention after an amniotomy is to a. Assess the color of the amniotic fluid. b. Change the patient's gown. c. Estimate the amount of amniotic fluid. d. Assess the fetal heart rate.

ANS: D Feedback A This is important, but not the top priority. B This is important for patient comfort, but it is not the top priority. C This is not a top priority for this patient. D The fetal heart rate must be assessed immediately after the rupture of the membranes to determine whether cord prolapse or compression has occurred.

12. When the mother's membranes rupture during active labor, the fetal heart rate should be observed for the occurrence of which periodic pattern? a. Increase in baseline variability b. Nonperiodic accelerations c. Early decelerations d. Variable decelerations

ANS: D Feedback A This is not an expected occurrence after the rupture of membranes. B Accelerations are considered reassuring; they are not a concern after rupture of membranes. C Early declarations are considered reassuring; they are not a concern after rupture of membranes. D When the membranes rupture, amniotic fluid may carry the umbilical cord to a position where it will be compressed between the maternal pelvis and the fetal presenting part, resulting in a variable deceleration pattern.

18. The most likely interpretation of an elevated immunoglobulin M (IgM) level in a newborn is a. The infant was breastfed during the first hours after birth b. Transference of immune globulins from the placenta to the infant c. An overwhelming allergic response to an antigen d. A recent exposure to a pathogenic agent

ANS: D Feedback A This is the IgA. B This is the IgG. C This is not associated with elevated levels of IgM. D An elevated level of IgM is associated with exposure to infection in utero because IgM does not cross the placenta.

19. When using IA for FHR, nurses should be aware that a. They can be expected to cover only two or three patients when IA is the primary method of fetal assessment. b. The best course is to use the descriptive terms associated with EFM when documenting results. c. If the heartbeat cannot be found immediately, a shift must be made to electronic monitoring. d. Ultrasound can be used to find the fetal heartbeat and reassure the mother if initial difficulty was a factor.

ANS: D Feedback A When used as the primary method of fetal assessment, auscultation requires a nurse-to-patient ratio of one to one. B Documentation should use only terms that can be numerically defined; the usual visual descriptions of EFM are inappropriate. C Locating fetal heartbeats often takes time. Mothers can be reassured verbally and by the ultrasound pictures if that device is used to help locate the heartbeat. D Locating fetal heartbeats often takes time. Mothers can be reassured verbally and by the ultrasound pictures if that device is used to help locate the heartbeat.

1. As an adjunct to inductions, a number of procedures to ripen the cervix are employed. One of these methods is the vaginal administration of preparations using prostaglandins. Before administering this medication, the nurse should be aware that this class of drug is an appropriate choice for women who have had a prior cesarean birth. Is this statement true or false?

ANS: F Prostaglandins are contraindicated in patients who have had a prior cesarean birth or other uterine surgery. A side effect of prostaglandin administration is hyperstimulation of the uterus. This may result in reduced uterine blood flow to the fetus, impaired gas exchange, and increased risk of uterine rupture. Prostaglandins should also be used with caution in women who have asthma, heart disease, glaucoma, or renal or hepatic dysfunction.

1. The Joint Commission has recognized that pain management is an essential part of care in all health care settings. Since many women may desire an un-medicated birth, the pain management standards set by The Joint Commission do not apply in a labor setting. Is this statement true or false?

ANS: F The nurse must still evaluate the patient's level of pain and discuss nonpharmacologic comfort interventions that may be part of her birth plan. The patient has the right to effective pain management and must be competently and routinely assessed and managed. The woman may also change her plan for pain management as her labor progresses. Education of the woman and her family regarding postpartum pain control must also be included in her discharge teaching.

1. Occasionally a woman arrives at the intrapartum unit ready to give birth. Bearing down, grunting, or stating something like "the baby's coming" should direct the nurse to advise the client, "Do not push, pant, and blow until the physician arrives." Is this statement true or false?

ANS: F The nurse's priority is to prevent or reduce injury to mother and infant if delivery is imminent. The emergency delivery kit should be obtained and preparation made for immediate delivery. An abbreviated assessment should be completed in order to obtain the mother's name, that of the support partner, and her care provider. Estimated date of delivery, allergies, and prenatal care are also important information. If time allows, maternal vital signs should be done, as well as a fetal assessment. After delivery, the priority is to maintain the infant's airway and temperature.

1. Part of the newborn assessment includes examination of the umbilical cord. The cord should contain 2 vessels: one vein and one artery. Is this statement true or false?

ANS: F The umbilical cord contains 3 vessels: two small arteries and one large vein. A 2-vessel cord may be an isolated abnormality or it may be associated with chromosomal and renal defects.

2. The woman in labor should be encouraged to use the Valsalva maneuver (holding one's breath and tightening abdominal muscles) for pushing during the second stage. Is this statement true or false?

ANS: F The woman should actually be discouraged from using the Valsalva maneuver. This activity increases intrathoracic pressure, reduces venous return, and increases venous pressure. During the Valsalva maneuver, fetal hypoxia may occur. The process is reversed when the woman takes a breath.

1. According to the 2008 National Institute of Child Health and Human Development (NICHD) nomenclature, there are four categories of fetal heart rate variability: absent, minimal, moderate, and marked. Is this statement true or false?

ANS: T Absent: the amplitude range is undetectable. Minimal: detectable to less than or equal to 5 bpm. Moderate (normal): 6 to 25 bpm. Marked: a range greater than 25 bpm.

2. In many facilities protocols allow the nurses to obtain transcutaneous bilirubin measurements (TcB) using a bilirubin meter, without the order of a nurse practitioner or physician. Is this statement true or false?

ANS: T Bilirubinometers are non-invasive devices to measure bilirubin levels in the infant's skin, thus avoiding repeated skin punctures to obtain blood samples. Abnormal results of TcB be should be confirmed with a total serum bilirubin (TsB). The National Association of Neonatal Nurses recommends obtaining a TcB or TsB on all infants prior to discharge.

2. The use of hydrotherapy is an important nonpharmacologic modality to promote relaxation in labor. The nurse should offer the woman in labor a warm bath or shower (if no contraindications exist). The mild nipple stimulation that occurs in the water may intensify her contractions. Is this statement true or false?

ANS: T Especially in a woman whose labor has slowed, the nipple stimulation will cause her posterior pituitary gland to secrete oxytocin. The release of this hormone may intensify labor contractions.

2. The birth attendant evaluates whether labor and birth are safer for the woman and her fetus than continuing the pregnancy. The Bishop Scoring System remains a popular tool to assist in this task. Is this statement true or false?

ANS: T The Bishop score uses 5 factors to determine readiness for labor (dilation, effacement, consistency, position, and fetal station). The likelihood of a vaginal birth is similar to that of spontaneous labor if the score is greater than 8.


Conjuntos de estudio relacionados

World Regional Studies Semester 2 Final

View Set

Key Terms Ch. 7:5 Muscular System

View Set

Chapter 9: Grounds Upon Which a Contract May Be Set Aside: Mistake and Misrepresentation

View Set

Oxford ATPL - Human Performance P2

View Set

Nursing 158 contraception Final exam

View Set

CHAPTER 7: ANTIBACTERIAL DRUGS THAT Disrupted the BACTERIAL CELL WALL

View Set